Calculate the dimensions of an irregular hexagon whose perimeter in 98 cm. Answer will vary.

Answers

Answer 1

The dimensions of the irregular hexagon are 17 cm, 15 cm, 14 cm, 13 cm, 22 cm and 17 cm

Calculating the dimensions of the irregular hexagon

From the question, we have the following parameters that can be used in our computation:

Shape = irregular hexagon

Also, we have

Perimeter = 98 cm

The perimeter of a shape is the sum of its side length

An irregular hexagon has six unequal sides

using the above as a guide, we have the following:

17 + 15 + 14 + 13 + 17 + 22 = 98

This means that the side lengths are 17 cm, 15 cm, 14 cm, 13 cm, 22 cm and 17 cm

Read more about Perimeter at

https://brainly.com/question/24571594

#SPJ1


Related Questions

Round to the nearest given place.
1.45169 thousandths

Answers

Answer:

1.452

Step-by-step explanation:

1.45169 rounded to the thousandths place would be 1.452

The amount of time a certain brand of light bulb lasts is normally distributed with a mean of 2000 hours and a standard deviation of 25 hours. Out of 665 freshly installed light bulbs in a new large building, how many would be expected to last between 2030 hours and 2060 hours, to the nearest whole number?

Answers

To determine the number of light bulbs expected to last between 2030 hours and 2060 hours, we need to calculate the z-scores corresponding to these values and then use the z-score formula to find the proportion of light bulbs within this range.

The z-score formula is given by:

z = (x - μ) / σ

where:

x = value

μ = mean

σ = standard deviation

For 2030 hours:

z1 = (2030 - 2000) / 25

For 2060 hours:

z2 = (2060 - 2000) / 25

Now, we can use the z-scores to find the proportions associated with each value using a standard normal distribution table or calculator. The table or calculator will provide the area/proportion under the normal curve between the mean and each z-score.

Let's calculate the z-scores and find the proportions:

z1 = (2030 - 2000) / 25 = 1.2

z2 = (2060 - 2000) / 25 = 2.4

Using a standard normal distribution table or calculator, we can find the proportions corresponding to these z-scores:

P(z < 1.2) ≈ 0.8849

P(z < 2.4) ≈ 0.9918

To find the proportion of light bulbs expected to last between 2030 hours and 2060 hours, we subtract the cumulative probabilities:

P(2030 < x < 2060) = P(z1 < z < z2) = P(z < z2) - P(z < z1)

P(2030 < x < 2060) ≈ 0.9918 - 0.8849

Finally, we multiply this proportion by the total number of light bulbs (665) to get the estimated number of light bulbs expected to last between 2030 hours and 2060 hours:

Number of light bulbs ≈ (0.9918 - 0.8849) * 665

Rounding to the nearest whole number, the expected number of light bulbs that would last between 2030 hours and 2060 hours is approximately 71.[tex]\huge{\mathfrak{\colorbox{black}{\textcolor{lime}{I\:hope\:this\:helps\:!\:\:}}}}[/tex]

♥️ [tex]\large{\textcolor{red}{\underline{\mathcal{SUMIT\:\:ROY\:\:(:\:\:}}}}[/tex]

You purchase a tarp to cover the driveway when it snows. The
dimensions of your driveway are 10.2 ft. by 15.7 ft. If the tarp covers
your entire driveway, how many square feet are covered? Your answer
should be a number only. Do not round.

Answers

If the dimensions of your driveway are 10.2 ft. by 15.7 ft and the tarp covers your entire driveway,  the square feet are covered is [tex]160.14ft^{2}[/tex]

How can the dimension be calculated?

In mathematics, a dimension is the length or width of an area, region, or space in one direction. It is just the measurement of an object's length, width, and height.

With the given conditions,  we can formulate the expression as

;10.2 ft. * 15.7 ft

=160.14

Learn more about dimensions at;

https://brainly.com/question/26740257

#SPJ1

Suppose there are 17 jelly beans in a box-2 red, 3 blue, 4 white, and 8 green. What part of the jelly beans is blue? As a decimal rounded to the nearest ten-thousandth (four decimal places)

Answers

Blue Jelly beans are 0.1764 part of total .

Given,

Total beans = 17

Blue = 3

Red =2

White =4

Green =8

Now,

Out of total , green jelly beans = 8/17

Out of total , red jelly beans = 2/17

Out of total , white jelly beans = 4/17

Out of total , blue jelly beans = 3/17

Hence the blue jelly beans are 0.1764 part of total jelly beans .

Know more about decimal,

https://brainly.com/question/8985071

#SPJ1

Express 75 as a product of its prime factors write the prime factors in ascending order and give your answer in index form

Answers

Step-by-step explanation:

75 = 3 x 5 x 5    in prime factorization

Answer:

Step-by-step explanation:

3x5x5

3) Last year the mean salary for professors in a particular community college was $62,000 with a standard deviation of $2000. A new two year contract is negotiated. In the first year of the contract, each professor receives a $1500 raise.

Find the mean and standard deviation for the first year of the contract.
b) In the second year of the contract, each professor receives a 3% raise based on their salary during the first year of the contract. Find the mean and the standard deviation for the second year of the contract.

Answers

a) Mean for the first year of the contract: $63,500

The standard deviation for the first year of the contract: $2,000.

b) Mean for the second year of the contract: $65,405.

The standard deviation for the second year of the contract: $60.

We have,

To find the mean and standard deviation for the first year of the contract, we can use the given information and the properties of the normal distribution.

Given:

The mean salary for professors in the previous year = $62,000

Standard deviation in the previous year = $2,000

Raise in the first year = $1,500

Mean for the first year of the contract:

The mean salary for the first year can be obtained by adding the raise to the previous mean:

Mean = Previous Mean + Raise

Mean = $62,000 + $1,500

Mean = $63,500

The standard deviation for the first year of the contract:

Since each professor receives the same raise, the standard deviation remains the same:

Standard Deviation = $2,000

Therefore, for the first year of the contract, the mean salary is $63,500, and the standard deviation remains $2,000.

Now,

In the second year of the contract, each professor receives a 3% raise based on their salary during the first year of the contract.

To find the mean and standard deviation for the second year, we can use the given information and the properties of the normal distribution.

Mean for the second year of the contract:

To calculate the mean for the second year, we need to add a 3% raise to the mean salary of the first year:

Mean = Mean of the first year + (3% * Mean of the first year)

Mean = $63,500 + (0.03 * $63,500)

Mean = $63,500 + $1,905

Mean = $65,405

The standard deviation for the second year of the contract:

Since each professor receives a raise based on their salary from the first year, the standard deviation also increases. To calculate the standard deviation, we multiply the standard deviation from the first year by the percentage increase:

Standard Deviation = Standard Deviation of the first year * (Percentage Increase / 100)

Standard Deviation = $2,000 * (3 / 100)

Standard Deviation = $2,000 * 0.03

Standard Deviation = $60

Therefore, for the second year of the contract, the mean salary is $65,405, and the standard deviation is $60.

Thus,

a) Mean for the first year of the contract: $63,500

The standard deviation for the first year of the contract: $2,000.

b) Mean for the second year of the contract: $65,405.

The standard deviation for the second year of the contract: $60.

Learn more about mean here:

https://brainly.com/question/23263573

#SPJ1

546, 400 and 4,856 The value of 4 in which number is how many times larger than the value of 4 in which number.​

Answers

To determine how many times larger the value of 4 is in the second number compared to the first number, we need to calculate the ratio of the values.

First number: 546
Second number: 4,856

In the first number, the value of 4 is the same as the digit itself since it appears once.

In the second number, the value of 4 is larger since it appears twice.

To find the ratio, we divide the value of 4 in the second number by the value of 4 in the first number:

Value of 4 in second number: 2
Value of 4 in first number: 1

Ratio: 2/1 = 2

Therefore, the value of 4 in the second number is two times larger than the value of 4 in the first number.

I NEED HELP WITH STATISTICS

Answers

(a) The null hypothesis is that the mean birth weight of babies born at full term is 7.2 pounds. The alternative hypothesis is that the mean birth weight of babies born at full term is greater than 7.2 pounds.

(b) If the scientist decides to reject the null hypothesis, she might be making a Type I error.

(c) A Type II error occurs when the null hypothesis is false, but the scientist fails to reject it.

How to explain the information

a A Type I error occurs when the null hypothesis is true, but the scientist rejects it. In this case, the null hypothesis is that the mean birth weight of babies born at full term is 7.2 pounds. If the scientist rejects this hypothesis, she is saying that she believes that the mean birth weight is greater than 7.2 pounds. However, if the null hypothesis is true, then the mean birth weight is actually 7.2 pounds, and the scientist has made a mistake.

b In this case, the scientist would fail to reject the null hypothesis and conclude that the mean birth weight of babies born at full term is 7.2 pounds. However, the true mean birth weight is 7.7 pounds, so the scientist would be making a Type II error.

c In the context of a Type II error, suppose the null hypothesis is false, meaning there is indeed a significant difference or relationship. However, due to various factors such as insufficient sample size, low statistical power, or other limitations, the scientist fails to reject the null hypothesis. Consequently, they accept the null hypothesis even though it is false, leading to a Type II error.

Learn more about hypothesis on

https://brainly.com/question/606806

#SPJ1

You spin the spinner once. 123 What is P(less than 2)? Write your answer as a fraction or whole number.

Answers

Answer:

See below

Step-by-step explanation:

Since the spinner has the numbers 1, 2, and 3 on it, and we want to find the probability of spinning a number less than 2, there is only one possible outcome that satisfies this condition, which is spinning a 1. Therefore, the probability of spinning a number less than 2 is:

P(less than 2) = P(1) = 1/3

So the probability of spinning a number less than 2 is 1/3.

I NEED HELP WITH STATISTICS

Answers

Am here for you so need anything don’t message me

A scientist mixes water (containing no salt) with a solution that contains 35% salt. She wants to obtain 140 ounces of a mixture that is 15% salt. How many
ounces of water and how many ounces of the 35% salt solution should she use?

Answers

Answer:

.35x = 140(.15)

.35x = 21

x = 60 oz of 35% salt.

The scientist will need 60 oz of the 35% salt solution and 80 oz of water.

22% of what number is 3300

Answers

To find the number that corresponds to 22% of a given value, you can divide the given value by 22% (or 0.22).

Let's use this approach to find the number:

3300 ÷ 0.22 = 15,000

So, 22% of 15,000 is equal to 3300.

Answer:

x = 15000

Step-by-step explanation:

If you are using a calculator, simply enter 3300×100÷22, which will give you the answer.

(q11) Find the center of mass of the system of objects that have masses 2 , 3 and 5 at the point (-1,2),(1,1) and (3,3) respectively.

Answers

The center of mass of the system is approximately (3.7, 2.6).

The center of mass of a system of objects is the point where all the weight of the system appears to be concentrated. It can be defined as the average location of the weighted parts of the system.

The center of mass of a system is dependent on the mass of the objects in the system and their positions.

Let's determine the center of mass of the system with masses of 2, 3, and 5 at the points (-1, 2), (1, 1), and (3, 3), respectively. Let's name the masses m1, m2, and m3, respectively, and the coordinates (x1, y1), (x2, y2), and (x3, y3).

The x-component of the center of mass is given by the formula:

x= (m1x1 + m2x2 + m3x3) / (m1 + m2 + m3)

The y-component of the center of mass is given by the formula:

y= (m1y1 + m2y2 + m3y3) / (m1 + m2 + m3)

By using the given values, let's calculate the x and y components of the center of mass:

x = (2 x -1 + 3 x 1 + 5 x 3) / (2 + 3 + 5) = 37/10 ≈ 3.7y

= (2 x 2 + 3 x 1 + 5 x 3) / (2 + 3 + 5)

= 26/10 = 2.6

To learn more about : mass

https://brainly.com/question/28916233

#SPJ8

line
A storage bin has the shape of a cylinder with a conical top. What is the volume of the storage bin if
its radius is r = 4.9 ft, the height of the cylindrical portion is h = 9.7 ft, and the overall height is
H = 16.3 ft?
Volume (to the nearest tenth)

Answers

Answer:

Step-by-step explanation:

To find the volume of the storage bin, we need to calculate the volumes of both the cylindrical portion and the conical top, and then add them together.

The volume of the cylindrical portion can be calculated using the formula:

V_cylinder = π * r^2 * h

where r is the radius and h is the height of the cylindrical portion.

Substituting the given values, we have:

V_cylinder = π * (4.9 ft)^2 * 9.7 ftV_cylinder ≈ 748.07 ft³ (rounded to two decimal places)

The volume of the conical top can be calculated using the formula:

V_cone = (1/3) * π * r^2 * H_cone

where r is the radius and H_cone is the height of the conical top.

The height of the conical top can be obtained by subtracting the height of the cylindrical portion from the overall height:

H_cone = H - h = 16.3 ft - 9.7 ft = 6.6 ft

Substituting the given values, we have:

V_cone = (1/3) * π * (4.9 ft)^2 * 6.6 ftV_cone ≈ 243.24 ft³ (rounded to two decimal places)

To find the total volume, we add the volume of the cylindrical portion and the volume of the conical top:

Total volume = V_cylinder + V_cone

Total volume ≈ 748.07 ft³ + 243.24 ft³

Total volume ≈ 991.31 ft³ (rounded to one decimal place)

Therefore, the volume of the storage bin is approximately 991.3 ft³ (rounded to the nearest tenth).

Thus the required volume is, 975.05  ft³

Given that,

radius = r = 4.9

Height of cylindrical potion = h = 9.7

Overall height = 16.3

Since,

total height = Height of the cylinder + height of the cone

Height of the cone = 16.3 - 9.7

                                = 6.6 m

Since we know that,

Volume of a cylinder = πr² h

⇒ π (4.9)²(9.7)

⇒ 731.29 ft³

Since we also know that

Volume of a cone = (1/3)πr² h

= 731.29/3

= 243.76  ft³

Volume of the bin = volume of cone + volume of cylinder

= 731.29 ft³  + 243.76  ft³

Hence the volume be,

= 975.05  ft³

To learn more about volume visit:

https://brainly.com/question/16860802

#SPJ1

I need the solution!!!!​

Answers

Solve for the first variable in one of the equations, then substitute the result into the other equation.

Point form :
(-4,0)

Equation form :
x = -4, y = 0

Problem
Find the equation of the line.
Use exact numbers.

Answers

The Equation of line is y= -3/2x + 60

From the graph we take two coordinates as (2, 0) and (0, 3)

We know the formula for slope

Slope= (Change in y)/ (Change in x)

Slope = (3-0)/ (0-2)

Slope= 3 / (-2)

Slope= -3/2

Now, Equation of line

y - 0 = -3/2 (x-  2)

y= -3/2x + 6

Thus, the Equation of line is y= -3/2x + 60.

Learn more about Slope here:

https://brainly.com/question/3605446

#SPJ1

How do you solve the question Deloitte signs a contract on December 1 to provide 40 days of advisory services with receipt of $20,000 due at the end of the contract. On December 31, 75% of the services have been completed.

Answers

As of December 31, Deloitte should recognize $15,000 as revenue for the advisory services completed.

To solve the given question, we need to determine the amount of revenue that Deloitte should recognize as of December 31, based on the percentage of services completed.

Here's how we can calculate it:

Calculate the total revenue for the contract:

Total revenue = $20,000

Determine the percentage of services completed:

Percentage of services completed = 75%

Calculate the revenue recognized as of December 31:

Revenue recognized = Percentage of services completed × Total revenue

= 75% × $20,000

= $15,000

Therefore, as of December 31, Deloitte should recognize $15,000 as revenue for the advisory services completed.

Learn more about revenue click;

https://brainly.com/question/29567732

#SPJ1

Find the measure of ∠F
.

Answers

Step-by-step explanation:

triangle EFG is an isosceles triangle

angle G

= 180°-58°

= 122° (adj. angles on a str. line)

angle F

= (180°-122°)÷2

= 29° (angles in a triangle)

The number of combinations of eight items taken three at a time can be written as

Answers

Answer: 8C3

Step-by-step explanation: You need to use Combinations for this. Out of 8, you need to select 3, so answer is 8C3.

Multiply three consecutive digits backwards starting from 8, and divide by 3 factorial

(8*7*6)/(3*2*1)

=56

Which is the equation of the given line in point-slope form?

y−0=−1(x−8)

y−0=1(x+8)

y=−x+8

y−8=−1(x+0)

Answers

Answer:

y = -x + 8

Step-by-step explanation:

Let's break down the equation step by step to understand it better.

The equation in point-slope form is given as:

y - y1 = m(x - x1)

In this case, we have:

y - 0 = -1(x - 8)

The point-slope form uses a specific point (x1, y1) on the line and the slope (m) of the line.

Here, the point (x1, y1) is (8, 0), which represents a point on the line. This means that when x = 8, y = 0. The graph has a point at (8, 0), which confirms this information.

The slope (m) is -1 in this equation. The slope represents the rate at which y changes with respect to x. In this case, since the slope is -1, it means that for every unit increase in x, y decreases by 1. The negative sign indicates that the line has a downward slope.

By substituting the values into the equation, we get:

y - 0 = -1(x - 8)

Simplifying further:

y = -x + 8

This is the final equation of the line in slope-intercept form. It tells us that y is equal to -x plus 8. In other words, the line decreases by 1 unit in the y-direction for every 1 unit increase in the x-direction, and it intersects the y-axis at the point (0, 8).

If the graph has points at (0, 8) and (8, 0), the equation y = -x + 8 accurately represents that line.

Que número estoy pensando si al multiplicarlo por 4 y luego de sumarle 16 obtengo 8?

Answers

Answer:-2

Step-by-step explanation:

x(4)+16=8

what is the greatest common factor of 97 and 24? what the answer

Answers

1

Because the number 97 is a prime number

Answer:

The greatest common factor (GCF) of two numbers is the largest number that divides evenly into both numbers. Since 97 is a prime number and 24 is not divisible by 97, the GCF of 97 and 24 is 1.

Determine the a) total annual cost, and b) cost per mile to the nearest cent.
1. Liz Nolan drove 34,500 miles last year. The total of fixed costs was $9,916 and of variable costs was
$4,897.

Answers

Answer:

total annual cost: 49313

cost per mile: 14 cents

Step-by-step explanation:

find total annual cost by adding everything up

find cost per mile by doing 4897/34500

cost/ miles

we use variable cost since the only thing that might change each year is the amount of miles they drive

fixed costs are fixed and don't change

QUESTION 1 1.1 1.2 1.4 Use the definition of the derivative (first principles) to determine f'(x) if f(x)=2x 1.3 Determine f'(x) from first principles if f(x)=9-x². Determine f'(x) from first principles if f(x)=-4x².​

Answers

Based on the functions given, it should be noted that the values will be 2, -2x and -8x.

How to calculate the value

Using the definition of the derivative, we have:

f'(x) = lim(h->0) [f(x + h) - f(x)] / h

= lim(h->0) [2(x + h) - 2x] / h

= lim(h->0) 2h / h

= lim(h->0) 2

= 2

Therefore, f'(x) = 2.

For f(x) = 9 - x²:

Using the definition of the derivative, we have:

f'(x) = lim(h->0) [f(x + h) - f(x)] / h

= lim(h->0) [9 - (x + h)² - (9 - x²)] / h

= lim(h->0) [9 - (x² + 2xh + h²) - 9 + x²] / h

= lim(h->0) [-2xh - h²] / h

= lim(h->0) (-2x - h)

= -2x

Therefore, f'(x) = -2x.

For f(x) = -4x²:

Using the definition of the derivative, we have:

f'(x) = lim(h->0) [f(x + h) - f(x)] / h

= lim(h->0) [-4(x + h)² - (-4x²)] / h

= lim(h->0) [-4(x² + 2xh + h²) + 4x²] / h

= lim(h->0) [-4x² - 8xh - 4h² + 4x²] / h

= lim(h->0) [-8xh - 4h²] / h

= lim(h->0) (-8x - 4h)

= -8x

Therefore, f'(x) = -8x.

Learn more about functions on

https://brainly.com/question/31878183

#SPJ1

please help! mathematicians

Answers

Answer:

1 < m < 4

Step-by-step explanation:

If the roots of function f(x) are not real, then the discriminant (the part under the square root sign) will be negative.

Set the discriminant less than zero and rewrite in standard form:

[tex]\begin{aligned}16-4m(-m+5)& < 0\\16+4m^2-20m& < 0\\4m^2-20m+16& < 0\\4(m^2-5m+4)& < 0\\m^2-5m+4& < 0\end{aligned}[/tex]

Factor the quadratic:

[tex]\begin{aligned}m^2-5m+4& < 0\\m^2-4m-m+4& < 0\\m(m-4)-1(m-4)& < 0\\(m-1)(m-4)& < 0\end{aligned}[/tex]

The leading coefficient of the quadratic m² - 5m + 4 is positive.

Therefore, the graph will be a parabola that opens upwards.

This means that the interval where the parabola is below the x-axis (negative) is between the zeros of the quadratic. Since the zeros are m = 1 and m = 4, the solution to the inequality is 1 < m < 4.

Therefore, the values of m for which the roots of function f(x) will be non-real are 1 < m < 4.

Edwin sells jars of jam for $1.90 each. Determine how many jars of jam Edwin needs to sell to break even if the variable cost per jar is $1.10 and fixed expenses are $35,700.00 per year.

Answers

Edwin needs to sell 44,625 jars of jam to break even.

To determine how many jars of jam Edwin needs to sell to break even, we'll calculate the breakeven point using the following formula:

Breakeven Point = Fixed Expenses / (Selling Price per Unit - Variable Cost per Unit)

Given information:

Selling Price per Unit (SP) = $1.90

Variable Cost per Unit (VC) = $1.10

Fixed Expenses = $35,700.00 per year

Plugging in the values into the formula:

Breakeven Point = $35,700 / ($1.90 - $1.10)

Breakeven Point = $35,700 / $0.80

Breakeven Point = 44,625 jars

Therefore, Edwin needs to sell 44,625 jars of jam to break even.

for such more question on breakeven point

https://brainly.com/question/30551452

#SPJ8


Minka pours 1/4 cup of milk on her oatmeal each day for 7

Answers

Assuming you want the amount of milk in 7 days, we can set up a multiplication problem. Given Minka pours 1/4 cup of milk in her oatmeal each day, we can multiply that by 7 days to find that:

1/4 = 0.25
0.25 • 7 = 1.75, or 1 3/4

By the end of 7 days, Minka pours 1 3/4 cups of milk into her oatmeal collectively.

Jessica needs to know how much water her new fish tank can hold:

A rectangular prism with a length of 8 inches, a width of 4 inches, and a height of 9 inches.

Determine the total volume of the fish tank.

Answers

The fish tank has a total volume of 288 inch³. As a result, Jessica's new fish tank has a capacity of 288 inch³ for water.

The volume of a rectangular prism can be calculated using the formula:

V = l x b x h..........(i)

where,

V ⇒ Volume

l  ⇒ length

b ⇒ width

h ⇒ height

From the question, we are given the values,

l = 8 inches

b = 4 inches

h = 9 inches

Putting these values in equation (i), we get,

V = 8 x 4 x 9

⇒ V = 288 in³

Therefore, the fish tank has a total volume of 288 inch³. As a result, Jessica's new fish tank has a capacity of 288 inch³ for water.

Learn more about the volume of rectangular prism on:

https://brainly.com/question/24284033

6 I need steps to know how we did it

Answers

Answer:

D

Step-by-step explanation:

the right triangle contains h , the horizontal leg and the sloping side which is the hypotenuse of the right triangle.

the horizontal leg is half the measure of the side of the square base.

horizontal leg = 8 ÷ 2 = 4

using Pythagoras' identity in the right triangle

the square on the hypotenuse is equal to the sum of the squares on the other 2 sides, that is

h² + 4² = 10² ( subtract 4² from both sides )

h² = 10² - 4² ( take square root of both sides )

h = [tex]\sqrt{10^2-4^2}[/tex]

Two homebuyers are financing $137,000 to purchase a condominium. They obtained a 15-year, fixed-rate loan with a rate of 5.05%. They have been given the option of purchasing up to four points to lower their rate to 4.81%. How much will the four points cost them?

$1,370
$1,730
$4,580
$5,480

Answers

The cost of four points is:4 x $1,370 = $5,480Thus, the four points will cost the homebuyers $5,480.

Points can help lower mortgage rates on fixed-rate loans. The concept of points, which are basically prepaid interest, is a little complicated.

Each point is worth one percent of the loan amount, and paying points can lower your interest rate by a certain amount, typically about one-eighth to one-quarter of a percentage point.

The cost of points in the given scenario can be found using the following steps:

The loan amount to purchase a condominium is $137,000. The homebuyers obtained a 15-year fixed-rate loan with a rate of 5.05%.

If the homebuyers opt for four points, their loan rate will decrease to 4.81%.

To figure out how much the points will cost the homebuyers, we must first determine the cost of one point. Since one point is equal to 1% of the loan amount, one point on a $137,000 loan is:1% of $137,000 = $1,370

To learn more about : cost

https://brainly.com/question/2292799

#SPJ8

Other Questions
A loan of R30 000, granted at an interest rate of 19,2% p.a. compounded monthly, is to be amortised by regular equal monthly payments of R800 and a final payment made one month after the last equal payment of R800 (F < R800). The first payment is made one month after the loan was granted. The 20th, 21st, 22nd and 23rd payments are missed. To compensate for the missed payments, the equal additional amounts, rounded to the nearest cent, that must be added to each of the remaining payments, from the twenty fourth onwards, in orderto amortise the loan in the same time period, A number of boys (is/are. absent today.) Company A has no debt and 12 million shares outstanding trading for $4,00 each. Company A intends to acquire Company B, that has no debt and 3 million shares outstanding trading for $15,00. Company A shareholders know that synergy gains will be $14 million, so they will only proceed with the acquisition if their shares increase in 6% with the acquisition.What is the maximum offer that shareholders of company A will make to company B shareholders (number of shares of Company A in exchange for 1 Company B share) QUESTION 2If a company's days sales in receivables is 32 days what is their receivables turnover?O a. 12.25xO b. 14.37xO c. 11.41xO d. 12.67x Do you agree or disagree with each statement and why?Sarah is conducting a marketing research project. She is not sure what questions to ask or what types of consumers she should talk to. Therefore, Sarah should begin by conducting quantitative research, in order to generate ideas about the topic before she begins. Generally, minerals, crops and timber are personal property both while attached to the land and when removed from the land. O True O False Kasmaie-MAT 266 SYNC A Summer 2022 Problem 4. (1 point) Determine whether the integral is divergent or convergent. If it is convergent, evaluate it. If not, state your answer as divergent. 1 dw Answer(s) submitted: (incorrect) Problem 5. (1 point) Determine whether the integral is divergent or convergent. If it is convergent, evaluate it. If not, state your answer as divergent. 3dx Answer(s) submitted: (incorrect) Problem 6. (1 point) Determine whether the integral is divergent or convergent. If it is convergent, evaluate it. If not, state your answer as divergent. Answer(s) submitted: (incorrect) The 2020 balance sheet of Osaka's Tennis Shop, Incorporated, showed $610,000 in the common stock account and $2.5 million in the additional paid-in surplus account. The 2021 balance sheet showed $650,000 and $3 million in the same two accounts, respectively. If the company paid out $610,000 in cash dividends during 2021, what was the cash flow to stockholders for the year? (Enter your answer in dollars, not millions of dollars, e.g., 1,234,567.) what happens when the quantity of a good supplied at a given price is greater than the quantity demanded?excess supplystable pricesexact equilibriumincreased production a 250 gram sample of water at the boiling point had 35.0 kj of heat added. how many grams of water were vaporized? heat of vaporization for water is 40.6 kj/mole. Sara was trying to explain to her colleagues in the medical company where she is working that a corporate culture in which conformity to a statement of values andprinciples rather than simple obedience to laws and regulations is the prevailing model for ethical behavior. In this case, Sara is explaining which of the following:O A. Values-based culturesO B. Code of conductO C. Mission statementO D. Compliance-based cultures Which refers to an unjustified negative or harmful action toward a member of a group simply because the person belongs to that group? CASE 4An organization is working by grouping similar related jobs various departments. The HR department is not in direct touch with the Marketing Department and this has created problems. The HR head feels that he only knows about the HR department and similar is the case with the Marketing department. However, the truth is both of them have limited and specific skills.QUESTIONS Which type of organization structure does this organization has? 2 Marks1. What are the disadvantage of this type of organization structure? 2 Marks2. What will be the outcome of this mode of thinking in the near future for the organization? 1) The design of a document can directly reflect on the credibility of the writer. True/False 2) _____are the "design language" and expectations for how a certain type of document will look. 3) Which of the following is NOT an advantage of headings? a. They break up the page, making it look less formidable and more interesting. b. They enable your reader to see how the document is organized at a glance. Tc. hey help your reader turn quickly to sections of special interest. d. They make a document shorter. For C in terms of a parity-check matrix H of C. 5.4 Without using any of the bounds discussed in this chapter, show that (a) A(6, 5) = 2, (b) A (7,5) = 2. (Hint: For (a), first show that A(6, 5) 2 by producing a code explicitly. Then try to show that A (6, 5) 2 using a simple combinatorial argument similar to the one in Example 5.2.5.) Moziah decided to grow his business by acquiring funds through either loans or through issuing shares and hired you as a financial consultant. Explain the advantages and disadvantages of using equity capital and debt capital to finance a small business's growth and give Moziah your personal recommendation in his case. Question 66 Profit is defined as total revenue (TR) O plus total cost (TC) O times total cost (TC) O divided by total cost (TC) O minus total cost (TC) Question 67 A good example of an explicit cost of production is O both a and c O lease payments for the land on which a firm's factory stands O cost of forgone labor earnings for an entrepreneur O lost opportunity to invest in capital markets when the money is invested in one's business Which of the following situations doesnt illustrate the concept of cognitive dissonance?Question 2 options:a. You are required to wear a suit to work. You only own one suit.b. The advertising firm that you work for has just landed a new account representing a steakhouse. You dont eat meat for religious reasons.c. Your boss has just asked you to lie for her.d. You believe that Wal-Mart exploits their workers. You work for a jewelry designer and manufacturer that sells all of their product to Wal-Mart. Read in full, part IIIB of the Bzdziuch decision on damages. Identify the FALSE statement: Select one: a. The plaintiff claimed $200,000 in general damages and cost of future care. b. At the time of the injury, the plaintiff was 77 years old. c. Alberta Health Services did not submit a claim.d. In support of their damages' calculation, the defendants relied on cases that were not from Alberta. You take out a loan in the amount of $269,000 with annual equal repayments over the next 20 years. What is the balance of the loan after the 5th payment?